LSAT and Law School Admissions Forum

Get expert LSAT preparation and law school admissions advice from PowerScore Test Preparation.

User avatar
 Dave Killoran
PowerScore Staff
  • PowerScore Staff
  • Posts: 5862
  • Joined: Mar 25, 2011
|
#41325
Complete Question Explanation
(The complete setup for this game can be found here: lsat/viewtopic.php?t=8467)

The correct answer choice is (C)

The Local conditions in the question stem establish the following partial setup:

pt2_o91_g2_q12a.png

Because L, M, and N cannot live on the second, third, or fourth floors, they must live on either the first or fifth floors, and because the MN block and L each require a floor of its own, it can be inferred that L, M, and N will completely occupy the first and fifth floors, creating a L/MN dual split-option:

pt2_o91_g2_q12b.png

Given this information, you can Hurdle the UncertaintiesTM and come to the realization that only floor three and floor four are available for J and Q, and since Q cannot live on the same floor as O, Q must live on the third floor and thus J must live on the fourth floor, resulting in the following setup:

pt2_o91_g2_q12c.png

It follows that answer choice (C) is correct. Also of interest is the fact that this is the first question in the game to directly address the OQ not-block. The test makers probably left this rule out of the game until the end in an effort to see whether careless test takers would forget about the rule and then miss the question. You must fix the rules in your mind at the beginning of the game and never forget them!
You do not have the required permissions to view the files attached to this post.

Get the most out of your LSAT Prep Plus subscription.

Analyze and track your performance with our Testing and Analytics Package.